In a game, there are 12 identical balls of which seven are red and five are green.

Five red balls and two green balls have number ‘2’ written on them. The rest of the

red balls have number ‘1’ written on them, and the rest of the green balls have the

number ‘3’ written on them. A random sample of three balls is selected without

replacement. Let denotes the event that all the balls selected are red and

denotes that the sum of numbers of the three balls is equal to 6. Calculate:

(i) P(A) ,

(ii) P(B),

(iii)P ( A∩ B),

(iv)P(A|B).​

Answers

Answer 1

Answer:

its number 2 and if its a mutable answers writ 3 also

Answer 2

The probabilities are: (i) P(A) = 1/6

(ii) P(B) = 38/55

(iii) P(A ∩ B) = 1/110

(iv) P(A|B) ≈ 0.00152

To calculate the probabilities, let's first find the total number of ways to choose 3 balls out of the 12 balls.

Total number of ways to choose 3 balls out of 12 = 12C3 = (12 * 11 * 10) / (3 * 2 * 1) = 220

(i) P(A): Probability that all three balls selected are red.

Number of ways to choose 3 red balls out of 7 red balls = 7C3 = (7 * 6 * 5) / (3 * 2 * 1) = 35

P(A) = Number of favorable outcomes / Total number of outcomes = 35 / 220 = 1/6

(ii) P(B): Probability that the sum of the numbers on the three balls is equal to 6.

The possible combinations that sum up to 6 are: (2, 2, 2), (2, 2, 1), and (1, 1, 3).

Number of ways to choose 3 balls such that their sum is 6:

- For (2, 2, 2), we have 1 choice for each color, so 1 * 1 * 1 = 1 way.

- For (2, 2, 1), we have 1 choice for each color, so 1 * 1 * 1 = 1 way.

- For (1, 1, 3), we have 6 choices for the first red ball (all are labeled '1'), 5 choices for the second red ball (since one '1' is already taken), and 5 choices for the green ball labeled '3', so 6 * 5 * 5 = 150 ways.

Total number of ways to choose 3 balls with sum 6 = 1 + 1 + 150 = 152

P(B) = Number of favorable outcomes / Total number of outcomes = 152 / 220 = 38/55

(iii) P(A ∩ B): Probability that all three balls selected are red and the sum of their numbers is equal to 6.

From the above calculations, we know that there are 1 way to choose (2, 2, 2) and 1 way to choose (2, 2, 1) such that all three balls are red and the sum is 6.

P(A ∩ B) = Number of favorable outcomes / Total number of outcomes = 2 / 220 = 1/110

(iv) P(A|B): Probability that all three balls selected are red, given that the sum of their numbers is equal to 6.

P(A|B) = P(A ∩ B) / P(B) = (1/110) / (38/55) = (1/110) * (55/38) ≈ 0.00152 (rounded to five decimal places).

To know more about probabilities:

https://brainly.com/question/29381779

#SPJ2


Related Questions

What are the values of A, B, C, and D?
A = 0; B = 1; C = -3; D = 3
A = 0; B = 1; C = 3; D = -3
A = 1; B = 0; C = -3; D = 3
A = 1; B = 0; C = 3; D = -3
Don’t know real answer

Answers

A=1;b=0;c=3 d=23234422

The values of A, B, C and D are (d) A = 1, B = 0, C = 3 and D = -3

How to solve for A, B, C and D?

From the question, we have:

x^3 + 8x - 3 = Ax^3 + 5Ax + Bx^2 + 5B + Cx + D

Collect like terms

x^3 + 8x - 3 = Ax^3 + Bx^2 + 5Ax  + Cx + 5B + D

By comparing the coefficients, we have:

Ax^3 = x^3

Bx^2 = 0

5Ax + Cx = 8x

5B + D = -3

Remove the x factors

A = 1

B = 0

5A + C = 8

5B + D = -3

Substitute A = 1 in 5A + C = 8

5(1) + C = 8

Solve for C

C = 3

Substitute B = 0 in 5B + D = -3

5(0) + D = -3

Solve for B

D = -3

Hence, the values of A, B, C and D are (d) A = 1, B = 0, C = 3 and D = -3

Read more about partial fractions at:https://brainly.com/question/18958301

#SPJ6

Ms. Harlow had 56 tickets. She gave 2 tickets to each of her 21 students.
Which set of equations shows how many tickets Mrs. Harlow had left?
21 x 2 = 42
OA)
56 - 42 = 14
56 - 21 = 35
ОВ)
35 - 2 = 33
21 x 2 = 42
OC)
56 + 42 = 98
56 - 21 = 35
OD)
35 + 2 = 37

Answers

Answer: 56 - 42

Step-by-step explanation: She gave 2 to EACH of her 21 students. So 2 * 21 = 42. 56 - 42

In a random sample of 150 customers of a high-speed internet provider, 63 said that their service had been interrupted one or more times in the past month. Find a 95% confidence interval for the proportion of customers whose service was interrupted one or more times in the past month.

Answers

Answer:

The correct answer is "0.3410, 0.4990".

Step-by-step explanation:

Given values are:

[tex]n=150[/tex]

[tex]p=\frac{63}{150}[/tex]

  [tex]=0.42[/tex]

At 95% confidence interval,

C = 95%

z = 1.96

As we know,

⇒ [tex]E=z\sqrt{\frac{p(1-p)}{n} }[/tex]

By substituting the values, we get

       [tex]=1.96\sqrt{\frac{0.42\times 0.58}{150} }[/tex]

       [tex]=1.96\sqrt{\frac{0.2436}{150} }[/tex]

       [tex]=0.0790[/tex]

hence,

The confidence interval will be:

= [tex]p \pm E[/tex]

= [tex]0.42 \pm 0.079[/tex]

= [tex](0.3410,0.4990)[/tex]

(a+b)²=hihihihihihihihiihihihi

Answers

Answer:

(a+b)²=a²+b²+2ab

Step-by-step explanation:

The square of sum of two terms is equal to the squared plus squared plus times product of and . In mathematics, the plus whole squared algebraic identity is called in three ways. The square of sum of two terms identity.

Solve the equation for x in terms of c.
2/3(cx+1/2)-1/4=5/2

Answers

Answer:

Step-by-step explanation:

The answer you have chosen is not correct. Let's walk through the simplification process, shall we?

Begin by adding over the 1/4, after you find the common denominator, that is. 5/2 with a denominator of 4 is 10/4:

[tex]\frac{2}{3}(cx+\frac{1}{2})=\frac{11}{4}[/tex] and then multiply both sides by the reciprocal of 2/3:

[tex]cx+\frac{1}{2}=\frac{33}{8}[/tex] then subtract the 1/2 in the form of 4/8 (common denominator and all...) to get

[tex]cx=\frac{29}{8}[/tex] and finally divide both sides by c to get

[tex]x=\frac{29}{8c}[/tex]  That's choice C.

Is 9/54 equivalent to 1/3

Answers

Answer: No.

Step-by-step explanation:

9/54 is equal to 1/6

Answer:

no, it is not

Step-by-step explanation:

9/54 can be simplified by dividing both sides by 3, getting 3/18. you can do this again and get 1/6. significantly smaller that l/3

A rectangular Carrer has a perimeter of 240cm breadth of 50cm.What is it's length

Answers

Step-by-step explanation:

The perimeter of a rectangle is the length of all its 4 sides. Formula to calculate the perimeter of a rectangle is:

Perimeter of Rectangle = 2 × Length + 2 × Breadth

The perimeter can be represented using a model as below.

Perimeter = Length + Breadth + Length + Breadth

= 2 × Length + 2 × Breadth

Length + Breadth = Perimeter ÷ 2

find the area of the triangle 15cm, 18cm, 5cm

Answers

Answer:

38

Step-by-step explanation:

Answer:

[tex]32.62\:\mathrm{cm^2}[/tex]

Step-by-step explanation:

The area of a triangle with side lengths [tex]a[/tex], [tex]b[/tex], and [tex]c[/tex] is given by:

Let [tex]s=\frac{a+b+c}{2}[/tex] (semi-perimeter)

[tex]A=\sqrt{s(s-a)(s-b)(s-c)}[/tex]

We're given three side lengths with lengths 15 cm, 18 cm, and 5 cm. Therefore, the semi-perimeter, [tex]s[/tex], is equal to [tex]\frac{15+18+5}{2}=\frac{38}{2}=19[/tex].

Thus, the area of this triangle is:

[tex]A=\sqrt{19(19-18)(19-15)(19-5)},\\A=\sqrt{19\cdot 1\cdot 4\cdot 14},A=\sqrt{1064}=32.6190128606\approx \boxed{32.62\:\mathrm{cm^2}}[/tex]

An educational psychologist wondered whether there was a relationship between the amount of academic pressure a high school student felt and their plans for college. They surveyed a random sample of 300 high school students throughout the country about their college plans and whether they felt academic pressure. Here are the responses and partial results of a chi-square test (expected counts appear below observed counts):
Chi-square test:
Planned years of college vs. feeling academic pressure now
O years Up to 4 years Up to 4 years More than 4 years Total
Yes 31 48 161 240
31.2 48 160.8
No 8 12 40 60
7.8 12 40.2
Total 39 60 201 300
They want to use these results to carry out a xạ test of independence. Assume that all conditions for inference were met. What are the values of the test statistic and P-value for their test?
What are the values of the test statistic and P-value for their test?
A. x² = 0.002;
0.001 < P-value < 0.0025
B. x² = 0.002;
P-value > 0.25
C. x² = 0.007;
0.005 < P-value < 0.01
D. x² = 0.007;
P-value > 0.25

Answers

Answer:

D. x² = 0.007;

P-value > 0.25

Step-by-step explanation:

The observed and expected values are given below :

Yes____31 48 161 ____240

______31.2 48 160.8

No ____ 8 12 40_____ 60

______7.8 12 40.2

Total __ 39 60 201 ___300

The Chisquare statistic, χ²:

Σ(Observed - Expected)²/ Expected

Chi-Squared Values:

0.00128205 __ 0 __ 0.000248756

0.00512821 __ 0 __ 0.000995025

(0.00128205 + 0 + 0.000248756 + 0.00512821 + 0 + 0.000995025 )

= 0.007654041

The degree of freedom = (row - 1) * (column - 1)

Degree of freedom = (2-1)*(3-1) = 1*2= 2

The Pvalue = 0.9962

Pvalue > 0.25

On a coordinate plane, a line goes through (negative 3, negative 3) and (negative 1, 5). What is the equation of the line parallel to the given line with an x-intercept of 4?

Answers

y = mx + c

m = gradient

gradient of line:
[5 - (-3)]/[(-1) - (-3)]
= 8/2
= 4

y = mx + c
subsitute (4, 0)
0 = (4)(4) + c
0 = 16 + c
c = -16

equation of the line:

y = 4x - 16

hope this helped :)



Answer:

4, -16

Step-by-step explanation:

Find the area of the composite figure.

Answers

Answer:

25m²

Step-by-step explanation:

it might be helpful for you

what'd the greatest common factor (GCF) for each pair of numbers. 25, 55 The GCE IS​

Answers

Answer:

5

Step-by-step explanation:

5 l 25,55

   l 5,11

what is a bank statement​

Answers

Step-by-step explanation:

a printed record of the balance in a bank account and the amounts that have been paid into it and withdrawn from it, issued periodically to the holder of the account.

Consider the expression 63+81 how can you use the distributive property and the gcf to find an equivalent expression?explain how you can check your answer

Answers

Step-by-step explanation:

63+81

gcf = 9

63÷9=7, 81÷9=9

so, 63+81 = (9×7)+(9×9)

= 9×(7+9)

63+81 = 144

9x(7+9) = 9×16 = 144

Which is equivalent to f/125)*? O 125+ 0 1254 125" C. 125/11​

Answers

Answer:

I think choose (1)

[tex]125 ^{ \frac{1x}{3} } [/tex]

A box has two balls, one white and one red. We select one ball, put it back in the box, and select a second ball (sampling with replacement). Find the probability of the following events:
a. Let F = the event of getting the white ball twice.
b. Let G = the event of getting two balls of different colors.
c. Let H = the event of getting white on the first pick.
d. Are F and G mutually exclusive?
e. Are G and H mutually exclusive?

Answers

Answer:

See explanation

Step-by-step explanation:

Given

Represent the balls with the first letters

[tex]W =1[/tex]

[tex]R =1[/tex]

Solving (a): P(F) --- White balls twice

The event of F is:

[tex]F = \{(W,W)\}[/tex]

So:

[tex]P(F) = P(W) * P(W)[/tex]

[tex]P(F) = \frac{n(W)}{n} * \frac{n(W)}{n}[/tex]

[tex]P(F) = \frac{1}{2} * \frac{1}{2}[/tex]

[tex]P(F) = \frac{1}{4}[/tex]

Solving (b): P(G) --- two different colors

The event of G is:

[tex]G = \{(W,R),(R,W)\}[/tex]

So:

[tex]P(G) = P(W) * P(R) + P(R) * P(W)[/tex]

[tex]P(G) = \frac{n(W)}{n} * \frac{n(R)}{n} + \frac{n(R)}{n} * \frac{n(W)}{n}[/tex]

[tex]P(G) = \frac{1}{2} * \frac{1}{2} + \frac{1}{2} * \frac{1}{2}[/tex]

[tex]P(G) = \frac{1}{4} + \frac{1}{4}[/tex]

[tex]P(G) = \frac{1}{2}[/tex]

Solving (c): P(H) --- White picked first

The event of H is:

[tex]H = \{(W,R),(W,W)\}[/tex]

So:

[tex]P(H) = P(W) * P(R) + P(W) * P(W)[/tex]

[tex]P(H) = \frac{n(W)}{n} * \frac{n(R)}{n} + \frac{n(W)}{n} * \frac{n(W)}{n}[/tex]

[tex]P(H) = \frac{1}{2} * \frac{1}{2} + \frac{1}{2} * \frac{1}{2}[/tex]

[tex]P(H) = \frac{1}{4} + \frac{1}{4}[/tex]

[tex]P(H) = \frac{1}{2}[/tex]

Solving (d): F and G, mutually exclusive?

We have:

[tex]F = \{(W,W)\}[/tex]

[tex]G = \{(W,R),(R,W)\}[/tex]

Check for common elements

[tex]n(F\ n\ G) = 0[/tex]

Hence, F and G are mutually exclusive

Solving (e): G and G, mutually exclusive?

We have:

[tex]G = \{(W,R),(R,W)\}[/tex]

[tex]H = \{(W,R),(W,W)\}[/tex]

Check for common elements

[tex]n(G\ n\ H) = 1[/tex]

Hence, F and G are not mutually exclusive

What is the standard form of 14587.25​

Answers

Answer:

fourteen thousand five hundred and eighty seven point twenty five.

Step-by-step explanation:

The given number is 14587.25.

We need to write it in standard form.

7 is in ones place, 8 is in tens place, 5 is in hundreds place, 4 is in thousands place and 1 is in ten-thousands place.

So,

14587.25 = fourteen thousand five hundred and eighty seven point twenty five.

Question 7(Multiple Choice Worth 1 points)
(02.08 LC)
To follow appropriate safety procedures, what should soccer players wear?
Goggles
Helmets
Mouth pieces
Shin guards

Answers

Answer:

The answer is Mouth pieces

The piece of safety equipment that should be worn when painting a ceiling is safety goggles. The correct option is c.

What are safety goggles?

Safety goggles are worn on the eyes. They are worn to get safety from sun, wind, and dust. If we work in a factory or some mechanical work, then goggles are must save eyes.

Here, we have,

When working with the ceiling, it has a chance to damage your eyes with the plaster and paint, so wearing safety goggles is necessary.

Thus, the correct option is c. safety goggles, in regard to being worn when painting a ceiling.

Learn more about safety goggles, here:

brainly.com/question/17225935

#SPJ2

Complete question:

Which of these pieces of safety equipment should be worn when painting a ceiling

a knee guards

b helmet

c safety goggles

d mouth guard

The answer and how to find the answer

Answers

Answer:

x = 1.9

Step-by-step explanation:

Thanks to a theorem we can use this proportion

10.47 : 4.44 = 4.44 = x

x = (4.44^2)/10.47 = 1,882865 = 1.9

A bicycle shop owner offers five styles of mountain bikes for $450, $275, $675, $490, and $300. He wants to increase the mean price but keep the median price and range of prices the same. Suggest a new set of prices for the five styles

Answers

Answer:

275, 350, 450, 550, 675

Step-by-step explanation:

Arrange in order

275, 300, 450, 490, 675

range 275 to 675

median 450

mean 438

---------------------------

Raise 300 to 350

Raise 490 to 550

New set of prices

275, 350, 450, 550, 675

range 275 to 675     same

median 450     same

mean 460    increased

Answer:

One set of prices could be: {275,300,450,600,675}

Another set could be: {275,300,450,600,675}

There are many other solutions possible.

====================================================

Explanation:

A = {450, 275, 675, 490, 300}

B = {275, 300, 450, 490, 675}

Set A is the original set of values in the order they were given to you. Set B is the sorted version of set A from smallest to largest.

The mean is found by adding up the values and dividing by 5 (because there are five items in the set).

The mean is (275+300+450+490+675)/5 = 2190/5 = 438. The shopkeeper wants to increase the mean to something larger, but keep the median and range the same.

The median is the middle most number. In set B, we can see that is 450. So the median is 450. We want to keep the median the same at 450.

The range is the difference in min and max

range = max - min = 675-275 = 400

We want to keep the range at 400

---------------------------

There are a number of ways to increase the mean, while keeping the median and range the same.

Let's say we keep the min and max the same. In order to increase the mean, we need to increase the 490 (second largest value) to something larger. Let's bump that up to 600 for instance.

Recomputing the mean gets us

(275+300+450+600+675)/5 = 2300/5 = 460

The old mean was 438 and the new mean is now 460. The mean has increased. This is due to the larger price pulling on the mean to get the mean to increase.

The median is still 450 because it's still in the direct middle of set C

C = {275,300,450,600,675}

The range is still the same as well because we haven't changed the min and max.

---------------------------

So one possible set could be

C = {275,300,450,600,675}

We could also have

D = {275,400,450,500,675}

The difference is that the 300 bumped to 400, and the 600 dropped to 500. You should find that the median and range are the same, while the mean is 460.

There are many possible solutions here.

You are working as an office apprentice for the bksb Newcastle Arena. You are helping collect a customer satisfaction survey for the arena. Enter the following replies into the table below.

Answers

Answer:

Step-by-step explanation:

i need to see the steps for simplifying 3(m-5)+m​

Answers

Answer:

4m - 15

Step-by-step explanation:

a( x + y) = ax + ay

[tex]3( m - 5 ) + m\\\\3m - 15 + m \\\\4m - 15[/tex]

Answer:

4m-15

Step-by-step explanation:

Distrubite 3 through the parentheisis

3m-15+m

Collect like terms

4m-15

Receipt-of-goods discounts tend to be offered in cases where?​

Answers

The invoice is enclosed with the goods and arrives at the time that they do

I WILL MARK BRAINLIEST TO WHOEVER ANSWERS CORRECTLY FIRST

Answers

Answer:

2 liter = 2000 ml

2000/250 = 8 bottles                                                                                             Step-by-step explanation:

The labelled quantity has been rounded to the nearest ml
The actual quantity will be between 249.5000 and 250.4999 (7 sf)
This can also be written as 249.5 <= x < 250.5
Or as the interval [249.5, 250.5) which means from 249.5 (including 249.5) to 250.5 (excluding 250.5)
If the quantity was 250.5 it would round up to 251.

Solve:
n = 8
4
O n = 2
0 n = 12
Ô n = 24
0 n = 32

Answers

Answer:

1/4 n = 8

n = 8 * 4

n = 32

so option 4 i.e.

n = 32. is the ans

Answer:

n=32

Step-by-step explanation:

Given :

[tex]\frac{1}{4} n=8[/tex]

Now,

Value of n can be calculated as :

[tex]\frac{1}{4} n=8[/tex]

n=8×4

n=32

Therefore, option (D) is correct.

Given the quadriceps function ,y=ax2+bc+c what happens to the graph when "a" is a positive?

Answers

Answer:

When "a" is positive, the parabola opens upwards and the vertex is the minimum value.

Step-by-step explanation:

The top of the Boulder Dam has an angle of elevation of 1.2 radians from a point on the Colorado River. Measuring the angle of elevation to the top of the dam from a point 155 feet farther down river is 0.9 radi- ans; assume the two angle measurements are taken at the same elevation above sea level. How high is the dam?

Answers

Answer:

382.925 feets

Step-by-step explanation:

The solution diagram is attached below :

Converting radian measurement to degree :

radian angle * 180/π = degree angle

1.2 * 180/π = 68.755°

0.9 * 180/π = 51.566°

Height of dam is h:

Using trigonometry :

Tan θ = opposite / Adjacent

Tan 68.755° = h / x

h = x Tan 68.755° - - - (1)

Tan 51.566° = h / (155+x)

h = (155+x) tan 51.566° - - - (2)

Equate (1) and (2)

x Tan 68.755 = (155+x) Tan 51.566

x Tan 68.755 = 155tan 51.566 + x tan 51.566

x Tan 68.755 = 195.32311 + x Tan 51.566

x Tan 68.755 - x Tan 51.566 = 195.32311

x(tan 68.755 - tan 51.566) = 195.32311

x * 1.3120110 = 195.32311

1.3120110x = 195.32311

x = 195.32311 / 1.3120110

x = 148.87307

Using :

h = x Tan 68.755

h = 148.87307 * tan(68.755)

h = 382.92539

h = 382.925 feets

Police response time to an emergency call is the difference between the time the call is first received by the dispatcher and the time a patrol car radios that it has arrived at the scene. Over a long period of time, it has been determined that the police response time has a normal distribution with a mean of 9.6 minutes and a standard deviation of 2.3 minutes. For a randomly received emergency call, find the following probabilities. (For each answer, enter a number. Round your answers to four decimal places.)

a. between 5 and 10 min
b. less than 5 min
c. more than 10 min

Answers

Answer:

a) 0.5447

b) 0.0228

c) 0.4325

Step-by-step explanation:

Normal Probability Distribution:

Problems of normal distributions can be solved using the z-score formula.

In a set with mean [tex]\mu[/tex] and standard deviation [tex]\sigma[/tex], the z-score of a measure X is given by:

[tex]Z = \frac{X - \mu}{\sigma}[/tex]

The Z-score measures how many standard deviations the measure is from the mean. After finding the Z-score, we look at the z-score table and find the p-value associated with this z-score. This p-value is the probability that the value of the measure is smaller than X, that is, the percentile of X. Subtracting 1 by the p-value, we get the probability that the value of the measure is greater than X.

Normal distribution with a mean of 9.6 minutes and a standard deviation of 2.3 minutes.

This means that [tex]\mu = 9.6, \sigma = 2.3[/tex]

a. between 5 and 10 min

This is the p-value of Z when X = 10 subtracted by the p-value of Z when X = 5. So

X = 10

[tex]Z = \frac{X - \mu}{\sigma}[/tex]

[tex]Z = \frac{10 - 9.6}{2.3}[/tex]

[tex]Z = 0.17[/tex]

[tex]Z = 0.17[/tex] has a p-value of 0.5675

X = 5

[tex]Z = \frac{X - \mu}{\sigma}[/tex]

[tex]Z = \frac{5 - 9.6}{2.3}[/tex]

[tex]Z = -2[/tex]

[tex]Z = -2[/tex] has a p-value of 0.0228

0.5675 - 0.0228 = 0.5447 probability that a randomly received emergency call is between 5 and 10 minutes.

b. less than 5 min

p-value of Z when X = 5, which from item a), is 0.0228, so 0.0228 probability that a randomly received emergency call is of less than 5 minutes.

c. more than 10 min

1 subtracted by the p-value of Z when X = 10, which, from item a), is of 0.5675.

1 - 0.5675 = 0.4325

0.4325 probability that a randomly received emergency call is of more than 10 minutes.

What is the value of the expression 3m-4.2 If m equals 2.1

Answers

Answer:

2.1

Step-by-step explanation:

Given :

3m-4.2 where, m=2.1

Now,

3(2.1)-4.2

6.3-4.2

2.1

Answer is 2.1

Can you solve this problem

Answers

Answer:

x = 18

Step-by-step explanation:

8 x 18 - 3 = 141

The answer is to your question is 18
Other Questions
What word doesn't belong ages, horse, matter, side, wait Identify the literary form suggested by the clue.couplets and balladsfablelegendmythnoveltall talefairy talepoetry 749BC (2y +34)Okay Congruent angle pairs : Find value of x What is the value of x in the triangle?3/2Xhelp please read the sentence. During the epidemic, early warnings circulated widely. which correctly analyzes each words part of speech in the sentence Find the first six terms of the sequence.a1 = -5, an = an-1 + 8 need help with this Q&A 2. Si x representa la edad de Ana, cmo seescribe en simbolos "la mitad de la edad deAna"? its the last one:) please help. giving brainlist Why are the danzante figures such an important find?Fr help a brother out no links pls What is the next term in the sequence below?0.25, 0.75, 2.25, 6.75, A. 6.25 B. 10.25 C. 20.25 D. 60.75 Which of the following is a simple sentence?Susan did a dance whenever she scored a goal.Martin pumped his fists in the air because hed made the winning touchdownLorenzo skipped around the bases after he hit a home run.Alma celebrated her basket with a wide smile. Swadaylab arck, afatis)ne that obat 5. Eind the raken$10 what phase of mitosis is shown in the diagram..? (Please can anyone help me out0 Gerald bought a computer on the installment plan. The price was $1,560. He paid $82 a month for 24 monthsWhat did Gerald pay in finance charges?O $310O $408O $456O $620 Which of the following would NOT be considered a barrier to making healthy food choices at home? A. You believe that healthy food does not taste good B. Your family has a strict monthly budget for food C. Your parents do not include you in meal planning D. Your house is 10 miles from the closest grocery store Write an in Standard Folm given the following points:(-2, 6), (9,8) How do you get 21/2? The lumberjack pulls on the sled with 40 N at an angle of 30 degrees, pulling so the sled moves at a constant velocity. 1) What is the x component of the applied force? 2) What is the y component of the applied force? 3) If the loaded sled has a mass of 65 kg, what is the magnitude of the force of gravity? 4) What is the magnitude of the normal force acting on the sled? 5) What is the coefficient of friction between the snow and the sled?